The years 1917, 1937, 1956, 1968, 1979, and 1990 are all notable for the occurrence of both popular uprisings and nea...

heidiz on June 22, 2019

Please Help

Could someone please explain why A, B, C, and D are wrong?

Replies
Create a free account to read and take part in forum discussions.

Already have an account? log in

mwilson1615 on July 21, 2019

Same here, and more specifically with C?

Ravi on July 22, 2019

@heidiz and @mwilson1615,

Happy to help. Let's take a look.

The argument is a causal one. It's saying that heavy sunspot activity
causes a rise in positively charged ions, which in turn causes people
to become anxious and irritable. The argument then concludes that the
sunspot activity has caused the popular uprisings. However, there's a
problem. The stimulus fails to connect irritability to uprisings at
all, so there is a large correlation to causation flaw in this
argument.

The basic structure of the argument in the stimulus is that A causes B
and B causes C. Therefore, A caused D.

(A) says, "The ancient Greeks sometimes attempted to predict the
outcome of future events by watching the flight patterns of birds.
Since the events themselves often matched the predictions, the birds
were probably responding to some factor that also influenced the
events."

(A) is saying that two things were caused by a third thing. This is
different from the causal argument made in the stimulus, so it's out.

(B) says, "Martha, Sidney, and Hilary are the city's three most
powerful politicians, and all three graduated from Ridgeview High
School. Although Ridgeview never had a reputation for excellence, it
must have been a good school to have produced three such successful
graduates."

(B) is saying that one thing caused 3 things (the success of Martha,
Sidney, and Hilary), but it doesn't offer any evidence, and there is
no causal chain present, so (B) is out.

(C) says, "Unusually cold weather last December coincided with a rise
in fuel prices. When it is cold, people use more fuel to keep warm;
and when more fuel is used, prices rise. Therefore if prices are high
next winter, it will be the result of cold weather."

(C) applies the transitive property to a couple of principles and then
confuses the necessary and sufficient conditions for each other (if
it's cold, the prices rise; therefore, if prices rise, it will be
cold). This doesn't match, so it's out.

(D) says, "The thirty healthiest people in a long-term medical study
turned out to be the same thirty whose regular diets included the most
vegetables. Since specific substances in vegetables are known to help
the body fight disease, vegetables should be part of everyone's diet."

(D)'s argument arrives at a conclusion that says something should be
done. The argument in the stimulus arrived at a causal conclusion, so
these conclusions don't match. Thus, (D) is out.

(E) says, "Acme's most productive managers are consistently those who
occupy the corner offices, which have more windows than other offices
at Acme. Since people are more alert when they are exposed to abundant
natural light, the greater productivity of these managers is probably
at least in part a result of their working in the corner offices."

(E) has a causal chain, just as the stimulus does. (E) is the only one
of the answers with a causal chain, so it's the correct answer choice.

Does this make sense? Let us know if you have any more questions!